Divisibility and Primes Drill

You might also like

Download as pdf or txt
Download as pdf or txt
You are on page 1of 24

Divisibility and Primes Drill

Warm-up Problems

Problem 1
Create prime boxes for each of the following numbers: 24, 32, 13, 360, 4200

Problem 2
If n is a positive integer and n is divisible by 20, how many unique prime numbers
must be a factor of n?

A) 1
B) 2
C) 3
D) 4
E) 5

Problem 3
When x is divided by 20 or by 7 an integer results. Is x divisible by 70?

Problem 4
Is y divisible by 64?

1) y is divisible by 32

2) y is divisible by 18

Problem 5
If d is an integer, is d even?

1) 2d is an even integer

2) d/2 is an odd integer

Problem 6
If x is an integer, is x divisible by 15?
1) When x is divided by 30, the answer is not an integer

2) When x is divided by 6, the answer is an integer

Problem 7
How many factors greater than 1 do 120, 210, and 270 have in common?

A) 1
B) 3
C) 6
D) 7
E) 30

Problem 8
What is the greatest common factor of 24, 54, and 60?

A) 2
B) 4
C) 6
D) 8
E) 12

Problem 9
Company H distributed $4000 and 180 pencils evenly among its employees, with
each employee getting an equal amount of cash (no change) and number of pencils (none
broken). What is the maximum number of employees that could be in Company H?

A) 9
B) 10
C) 20
D) 40
E) 180

Problem 10
Ramon wants to cut a rectangular board into identical square pieces. If the board
is 18 inches by 30 inches, what is the smallest number of square pieces he can cut without
wasting any of the board?
A) 4
B) 6
C) 9
D) 12
E) 15

Problem 11
A “prime power” is an integer that has only one prime factor. For example, 5 = 5,
25 = 5  5, and 27 = 3  3  3 are all prime powers, while 6 = 2  3 and 12 = 2  2  3, are
not. Which of the following numbers is not a prime power?

A) 49
B) 81
C) 100
D) 121
E) 243

Problem 12
What is the least common multiple of 10, 12, and 16?

A) 120
B) 160
C) 240
D) 320
E) 400

Problem 13
Manhattan GMAT decides to give away a free set of books to every 6th person
who signs up for a class and a free class to every 100th person who signs up for a class.
What student will be the first person to get a free class and a free set of books?

A) 100th
B) 200th
C) 300th
D) 500th
E) 600th
Problem 14
Steven gets his car washed every 15 days and vacuums his car every 9 days. If
Steven washes his car and vacuums it on the same day one day in October, in what month
will he next vacuum and wash his car on the same day?

1) Steven washed and vacuumed his car together some day in October after
October 14th.

2) Steven vacuumed but did not wash his car on October 11th.

Problem 15
If x and y are positive integers, is x a factor of y?

1) y is a factor of x

2) x > y

Problem 16
If a and b are integers and a > b > 1, which of the following could not be a
multiple of a or b?

A) a-1
B) b+1
C) b-1
D) a+b
E) ab

Problem 17
If m and n are both prime numbers, what is m?

1) mn = 25

2) m + n = 10

Problem 18
If g and h are different positive integers, what is the greatest common factor of g
and h?
1) g and h are both prime

2) g = h – 1

Problem 19
If x is a positive integer, what is the least common multiple of 12, 15, and x?

1) x is a prime number

2) x < 7

Problem 20
abc is a three digit number where a is the hundreds digit, b is the tens digit, and c
is the units digit. Let &(abc)& = (2a)(3b)(5c). For example, &(203)& = (22)(30)(53) =
500. How many three digit numbers abc will give a prime number when the formula
&(abc)& is applied?

A) 0
B) 1
C) 2
D) 3
E) 9
Medium Problems

Problem 21
If n is an integer and n3 is divisible by 24, what is the largest number that must be
a factor of n?

A) 1
B) 2
C) 6
D) 8
E) 12

Problem 22
If x is divisible by 18 and y is divisible by 12, which of the following must be
true?
I. x + y is divisible by 6
II. xy is divisible by 48
III. x/y is divisible by 6

A) I only
B) II only
C) I and II
D) II and III
E) none of the above

Problem 23
If p is divisible by 7 and q is divisible by 6, how many factors greater than 1 must
pq have?

A) 1
B) 3
C) 6
D) 7
E) 8

Problem 24
If r is divisible by 10 and s is divisible by 9, how many factors greater than 1 must
rs have?

A) 1
B) 4
C) 11
D) 13
E) 15

Problem 25
If t is divisible by 12, what is the smallest possible integer value for a that could
t2
allow a NOT to be an integer?
2

A) 2
B) 3
C) 4
D) 5
E) 6

Problem 26
f and g are both positive integers. Is (f+g) divisible by 10?

1) f is divisible by 5 and g is divisible by 5

2) fg is divisible by 10

Problem 27
If a, b, and c are all integers divisible by 3 and a > b > c > 0, then which of the
following must be divisible by 3?

I. a+b+c
II. a-b+c
III. abc/9

A) I only
B) III only
C) I and II
D) I and III
E) I, II, and III

Problem 28
If a and b are both positive integers, is (a+b) divisible by 3?

1) (a+b)2 is divisible by 9

2) (a-b)2 is not divisible by 3

Problem 29
(2a )(3b)
If a, b, and c are all positive integers, is the fraction a whole number?
(5c)

1) ab = 12

2) c = 3

Problem 30
If 2ab + 10a = 50 – 2b2 and b ≠ -5, how many factors does the term (a+b)2 have?

A) 2
B) 3
C) 4
D) 5
E) 9
Challenge Problems

Problem 31
A car factory paints cars in a repeating pattern of red, blue, black, gray. If the
first car was painted red, what color will the 463rd car be painted?

A) Red
B) Blue
C) Black
D) Gray
E) Cannot be determined from the information given

Problem 32
Jason deposits money at the bank on a Tuesday and returns to take it out 100 days
later. What day of the week did Jason take out the money from the bank?

A) Monday
B) Tuesday
C) Wednesday
D) Thursday
E) Friday

Problem 33
Seven consecutive positive integers are each divided by 6 and their remainders are
then added together. What is the sum of the six remainders?

1) The smallest integer has a remainder that is greater than 0 and divisible by 5.

2) The median of the seven integers has a remainder that is greater than 0 and
divisible by 2.

Problem 34
If n > 2 and n4 – 5n2 + 4 = 20h, is h an integer?

1) n/4 is an integer

2) n/5 is an integer
Problem 35
If k and n are both integers and k > n > 0, is k!/n! divisible by 30?

1) k + n = 30

2) k - n = 6

Problem 36
x and h are both positive integers. When x is divided by 7, the quotient is h with a
remainder of 3. Which of the following could be equal to x?

A) 7
B) 21
C) 50
D) 52
E) 57

Problem 37
ab
a, b, c, and d are all positive integers. If = 3.7, which of the following must
cd
be true?

I. ab is divisible by 5
II. c+d is divisible by 5
III. if c is even, then d must be even

A) I only
B) II only
C) III only
D) I and II
E) II and III

Problem 38
When (x+y)2 is divided by 5, what is the remainder?

1) When x+y is divided by 5, the remainder is 3

2) When x is divided by 5, the remainder is 4 and when y is divided by 5, the


remainder is 4
Problem 39
When x is divided by 10 the answer is y with a remainder of 4. If x and y are both
positive integers, what is the remainder when x is divided by 5?

A) 0
B) 1
C) 2
D) 3
E) 4

Problem 40
When x is divided by 13 the answer is y with a remainder of 3. When x is divided
by 7 the answer is z with a remainder of 3. If x, y, and z are all positive integers, what is
yz
the remainder of ?
13

A) 0
B) 3
C) 4
D) 7
E) 10
Problem 1 Solution:
24 32 13 360 4200

2 2 2 3 2 2 2 2 2 13 2 2 2 2 2 2
3 3 5 3 5 5 7

Prime boxes are the heart of divisibility problems. The contents of a number’s
prime box will consist of the prime factorization of that number. While choosing
numbers can be an easy solution to answering some GMAT problems, more difficult
problems will require that you be able to understand how to use prime boxes effectively.

Problem 2 Solution: B

If n is divisible by 20, its prime box will contain at least 2, 2, and 5. Because the
problem asks for the number of unique prime factors of n, 2 and 5 are the only such
primes. The correct answer is B.

Problem 3 Solution: yes

The question asks us whether x is divisible by 70, or in other words, whether x’s
prime box contains 2, 5, and 7. The question then gives us two separate pieces of
information about x’s prime box. First we are told that x’s prime box must contain 2, 2,
and 5. Next we are told that x’s prime box must contain 7. By combining these two
pieces of information, you know that x’s prime box must contain 2, 2, 5, and 7. Problems
dealing with divisibility will often give you “extra” information. In this case, the extra
information (an extra 2 in x’s prime box) is not important to answering the question. As
long as there is one 2, one 5, and one 7 in x’s prime box, we know that x is divisible by
70.

Problem 4 Solution: E

This is a common “trap” question for divisibility. The question asks us whether y
is divisible by 64. In other words, we are asked whether there are six 2’s in y’s prime
box. Statement (1) tells us that y is divisible by 32 or that there are five 2’s in y’s prime
box. Statement (2) tells us that y is divisible by 18 or that there two 3’s and a 2 in y’s
prime box. Neither of these statements is sufficient on its own. You must be cautious
when you combine the two statements. If you were simply to combine the prime
numbers from the two statements you would get six 2’s and two 3’s. This would lead
you to conclude that both statements together are sufficient. Let’s consider just the 2’s in
y’s prime box. Statement (1) tells us that there must be at least five 2’s in the prime box,
and statement (2) tells us that there must be at least one 2 in the prime box. This is a case
of statement overlap, in the same way that x > 5 and x > 10 together tell us nothing more
than that x > 10. Our final prime box for y should contain five 2’s and two 3’s, which is
not enough to conclude that y is divisible by 64. The correct answer is E.

Problem 5 Solution: B

The question asks whether d’s prime box contain at least one 2. Statement (1)
tells us that when you multiply d by 2 the number is even. This is equivalent to
appending a 2 to d’s prime box. However, we do not know what was originally in d’s
prime box. Insufficient. Statement (2) tells us that when you divide d by 2, the number
is odd. This is equivalent to taking a 2 away from d’s prime box. Because a 2 was taken
away, you know that d’s original prime box contained one (and only one) 2 inside.
Therefore statement (2) is sufficient. The correct answer is B.

Problem 6 Solution: C

We can rephrase the question to ask: does x’s prime box contain at least one 3 and
one 5? Let’s start with Statement (2). Statement (2) tells us that x is divisible by 6. This
means that x’s prime box contains a 2 and a 3. However, it tells us nothing about whether
x is divisible by 5. Insufficient. Statement (1) tells us that when x is divided by 30, the
answer is not an integer. 30 = 2 3 5. Therefore, x’s prime box can not contain one 2,
one 3, and one 5. But we are not sure which one (or more) of these primes is not in x’s
prime box. Insufficient. Taking both statements together, we know that x’s prime box
must contain a 2 and a 3, but can not contain a 2, a 3, and a 5. Therefore, x’s prime box
does not have a 5, so x is not divisible by 15. The correct answer is C.

Problem 7 Solution: D

By making prime boxes for 120, 210, and 270, you can quickly identify that they
have 2, 3, and 5 in common. However, the factors that they share can consist of any
combination of these prime factors. There are 3 single prime factors (2, 3, 5), 3 factors
consisting of two primes (2 3 = 6, 2 5 = 10, 3 5 = 15), and 1 factor consisting of all
three prime factors (2 3 5 = 30). Therefore, there are 7 factors that these three numbers
share. The correct answer is D.

Problem 8 Solution: C
One way to find the greatest common factor of numbers is to break the numbers
into their prime boxes and find what prime factors are in all boxes. 24 = 2 2 2 3, 54 =
2 3 3 3, 60 = 2 2 3 5. 2 3 is in all three boxes, so 6 is the greatest common
factor. The correct answer is C.

Problem 9 Solution: C

When things are distributed evenly, this means that they are divided with no
leftovers. In the case of Company H, it is distributing two things: money and pencils. In
order to distribute these items evenly, the number of employees must be a factor of each
of the two numbers. $4000 could be evenly distributed among 1000 employees, but 180
pencils could not be evenly distributed. This problem then becomes one of finding the
greatest common factor (GCF) of the two numbers. 4000 = 2 2 2 2 2 5 5 5 and
180 = 2 2 3 3 5. These numbers have 2 2 5 in common, so 20 is the GCF. The
correct answer is C.

Problem 10 Solution: E

Cutting a rectangular board into square pieces means that we need to cut pieces
that are equal in length in width. Not wasting any of the board means that we need to
choose a number that divides evenly into both 18 and 30. Wanting the smallest number
of pieces means that we need to choose the largest possible squares. With these three
stipulations, we choose the largest possible number that divides evenly into 18 and 30, or
the greatest common factor, which is 6. This would give us 3 pieces going one way and 5
pieces going the other. 3 5 = 15 gives us 15 even squares of dimensions 6" x 6". The
correct answer is E.

Problem 11 Solution: C

Break down each of the numbers into its prime factors. 49 = 7 7, 81 =


3 3 3 3, 100 = 2 2 5 5, 121 = 11 11, and 243 = 3 3 3 3 3. Since 100 has both
2 and 5 as prime factors, it is not a prime power. The correct answer is C.

Problem 12 Solution: C
One method to find the least common multiple is to create the largest prime box
that contains the prime boxes of all the given numbers. 10’s prime box contains 2 and 5,
12’s prime box contains two 2’s and a 3, and 16’s prime box contains four 2’s. The
prime box containing all these numbers must contain four 2’s, a 3, and a 5. Multiplying
2 2 2 2 3 5 gives us 240. The correct answer is C.

Problem 13 Solution: C

Recognize that the 6th, 12th, 18th, etc. persons will receive free books and the
100th, 200th, 300th, etc. persons will receive a free class. These sequences are multiples of
the numbers 6 and 100, and the question asks us to find the first person that will be a
multiple of 6 and a multiple of 100. This is the least common multiple. The least
common multiple of 6 and 100 is 300. The correct answer is C.

Problem 14 Solution: C

Notice that there are two unique, repeating events that sometimes occur
simultaneously. One happens every 9 days, the other happens every 15 days. Counting
by 9’s and by 15’s tells you that on the 45th day, both events will occur. This is the least
common multiple of the two numbers. The problem tells us that the two events happened
on a day in October, but doesn’t tell us which day. If this happened on October 1, then
the next day this would happen, 45 days later, would be November 15. If this happened
on October 31, then the next day this would happen, 45 days later, would be December
15. This means that the answer must be either November or December. But in order to
know for sure which month it is, you would first need to find the date that is 45 days
before December 1 – October 17. 45 days after October 16 is November 30. 45 days
after October 17 is December 1. The kernel of information we are then looking for is: did
the two events happen before or after/on October 17? Statement (1) tells us that the two
events happened on the same day after October 14. This statement is insufficient.
Statement (2) tells us that the car was vacuumed but not washed on October 11. The
other days that the car must have been vacuumed in October would be October 2, 18, 25.
One of these days was when the car was also washed, but we do not know which one.
This statement is insufficient. Using statements (1) and (2) we can deduce that the car
was washed and vacuumed on either October 18 or 25. We do not know which day it
was, but we know that 45 days after either of these dates would be in December. The
statements together are sufficient. The correct answer is C.

Problem 15 Solution: B
Statement (1) tells us that y is a factor of x, but y could be equal to x, in which
case x would be a factor of y. If y = 5, then x could be 10, 15, 20, etc., (which are
multiples, but not factors, of y) but it could also be equal to y, and thus x and y would be
factors of one another. Statement (1) is insufficient. Statement (2) tells us that x is larger
than y. Factors must be smaller than or equal to the number they divide. Statement (2) is
sufficient. The correct answer is B.

Problem 16 Solution: C

Since a multiple must be equal to or larger than the number it is a multiple of,
answer choice C cannot be a multiple of a or b since it is smaller than both integers a and
b. You can also try choosing numbers to plug into the various expressions, where a is
larger than b. If you choose a = 3 and b = 2, you can eliminate all but choices C and D,
and by selecting a = 4 and b = 2, you can eliminate answer choice D. The correct answer
is C.

Problem 17 Solution: A

Knowing that both m and n are prime integers, we are looking for a value for m.
Statement (1) tells us that the product of m and n is 25. Since the prime factorization of
25 is 5 5, we know that both m and n are equal to 5. Statement (1) is sufficient.
Statement (2) tells us that the sum of m and n is equal to 10. 5+5 is equal to ten, but so is
3+7 (no other addends would consist of two prime integers). Statement (2) is
insufficient. The correct answer is A.

Problem 18 Solution: D

The question asks us what the greatest common factor of two different integers is.
Without giving us values for integers, we must be able to identify characteristics of their
factors. Statement (1) tells us that both integers are prime. Since both of the integers
have only one integer in their prime box and the integers are different from one another,
we know that the greatest common factor of these two different prime numbers is 1.
Statement (1) is sufficient. Statement (2) tells us that g is one larger than h. Consecutive
numbers have no number larger than 1 in common. Statement (2) is sufficient. The
correct answer is D.

Problem 19 Solution: B
The least common multiple is the smallest multiple a group of numbers share.
Since two of the numbers are known and one is unknown, we can determine that the least
common multiple of 12 and 15 is 60 and rephrase the question to ask for the least
common multiple of 60 and x. Statement (1) tells us that x is a prime number but does
not give us any clue as to which prime x is, and we will not be able to determine the least
common multiple. Statement (1) is insufficient. Statement (2) tells us that x < 7. Going
through each integer from 1 to 6, we note that each is a factor of 60, which means that no
matter what x is, it is a factor of 60. The least common multiple of 60 and x must be 60
and statement (2) is sufficient. The correct answer is B.

Problem 20 Solution: B

A prime number can have only one integer in its prime box. Because we can only
use one prime number from the set {2,3,5}, two of the digits must be 0 and the other must
be 1. The only three values of abc that would result in a prime number are 100, 010, and
001. All other numbers would result in more than a single prime number in the &(abc)&
prime box. However, only one of those three numbers (100) is a three digit number. The
correct answer is B.

Problem 21 Solution: C

Start by considering the relationship between n and n3. Because n is an integer,


for every number that is in n’s prime box, there must be three of those numbers in n3’s
prime box. This means that n3’s prime box must have prime numbers in multiples of 3.
If there is one 3 in n3’s prime box, then there must be at least three 3’s. Now let's look at
what the problem tells us. If n3 is divisible by 24, then n3’s prime box must contain at
least three 2’s and a 3. But since n3 is a cube, it must contain at least three 3’s. Therefore
n must contain at least one 2 and one 3, or 2 3 = 6. The correct answer is C.

Problem 22 Solution: A

Start by looking at x’s and y’s prime boxes. If x is divisible by 18, then its prime
box must contain 2, 3, and 3. If y is divisible by 12, then its prime box must contain 2, 2,
and 3. Now let’s look at each of the statements. Statement I asks us to look at x + y. We
know that x must be 18 times a number, or 2 3 3 A. y must be 12 times a number, or
2 2 3 B. Therefore, x + y = (2 3 3 A) + (2 2 3 B) = 2 3 (2A+3B).
Therefore, the sum of x and y must be divisible by 6. In other words, because x and y are
both multiples of 6, their sum must also be a multiple of 6.
Statement II asks us to look at xy. When multiplying numbers, you can combine
the primes from both numbers to form the prime box of the product. Therefore, xy’s
prime box would contain 2, 2, 2, 3, 3, and 3. Breaking down 48 into its primes, we see
that 48 = 2 2 2 2 3. What we know of xy’s prime box does not have enough 2’s to
guarantee that xy is divisible by 48.
Statement III asks us to look at x/y. Be careful about statements that ask you to
divide. Without knowing more about y, there is very little we can deduce about x/y. We
know from the prime boxes of x and y, that both x and y can be divided by 6, allowing the
fraction x/y to be reduced. But y could contain many more prime integers, causing x/y to
be a non-integer. For example, x could be 18 and y could be 144, causing x/y to be a
fraction that is less than one. Only statement I must be true. The correct answer is A.

Problem 23 Solution: D

Start by making prime boxes for p and q. p contains a 7, and q contains 2 and 3.
Multiplying p by q causes all of these factors to end up in pq’s prime box. pq must be
divisible by 2 3 7, but it also must be divisible by each of the unique prime factors (2,
3, and 7) and each pair of prime factors that can be multiplied together (2 3 = 6, 2 7 =
14, 3 7 = 21). This gives us seven unique factors of pq: 2, 3, 6, 7, 14, 21, and 42. The
correct answer is D.

Problem 24 Solution: C

Start by making prime boxes for r and s. r contains 2 and 5, and s contains two
3’s. Multiplying r times s causes all of these factors to end up in rs’s prime box. rs must
be divisible by 2 3 3 5, but it also must be divisible by each of the numbers that use
some subset of these factors. This means we can use the 2 or not (two options), we can
use the 5 or not (two options), and we can use either zero, one, or two 3’s (three options).
Multiplying these options together gives us 2 2 3 = 12 factors. Because one of these
factors is 1 and we have been asked for factors greater than 1, this gives an answer of 11.
You could also get the correct answer by listing out all of the factors: 2, 3, 5, 6, 9, 10, 15,
18, 30, 45, and 90. The correct answer is C.

Problem 25 Solution: D

If t is divisible by 12, than t2 must be divisible by 144 or 2 2 2 2 3 3.


Therefore, t2 must be able to be divided by 2 at least four times, so a must be at least 5.
The correct answer is D.

Problem 26 Solution: E
The question asks us if (f+g) is divisible by 10. Statement (1) tells us that f and g
are each divisible by 5. Mathematically, we can demonstrate this by saying that f = 5x
and g = 5y. Substituting this into the expression, we see that f+g = (5x+5y) = 5(x+y).
Therefore, (f+g) must be divisible by 5, but it is uncertain whether it is divisible by 10.
Substituting numbers, we can quickly see that (5+5) is divisible by 10, but (5+10) is not
divisible by 10. Statement (1) is insufficient. Statement (2) tells us that fg’s prime box
must contain a 2 and a 5, but does not tell us anything about the prime box of (f+g).
Substituting numbers, we see that (2 5)= 10 and (10 10)=100, but (2+5) is not divisible
by 10 while (10+10) is divisible by 10. Statement (2) is insufficient. Combining
statements (1) and (2) tells us that both numbers are divisible by 5 and that one of the two
numbers is divisible by 2 (since fg = 10), but we do not know for certain that both
numbers are divisible by 10. 5+10 = 15 while 10+10 = 20. Statements (1) and (2)
together are insufficient. The correct answer is E.

Problem 27 Solution: E

Since a, b, and c are all divisible by 3, a = 3x, b = 3y, c = 3z, where x > y > z > 0
and all are integers. Substituting these new numbers into each expression will tell us
whether it is divisible by 3. Statement I: a+b+c = 3x+3y+3z = 3(x+y+z). This number
must be divisible by 3. Statement II: a-b+c = 3x-3y+3z = 3(x-y+z). This number must
be divisible by 3. Statement III: abc/9 = (3x3y3z)/9 = (27xyz)/9 = 3xyz. This number
must be divisible by 3. The correct answer is E.
Note that this is a problem very similar to some even/odd problems that you see
on the GMAT. Even numbers are numbers that are divisible by 2. Looking at these 3
statements, we are reminded of the similar property that (E) + (E) + (E) = (E) and (E) -
(E) + (E) = (E), while multiplying three even numbers means that the product must be
divisible by 8. Recognizing this connection can help make both divisibility and even/odd
problems easier to process.

Problem 28 Solution: A

The question asks us whether (a+b) is divisible by 3. Statement (1) tells us that
2
(a+b) is divisible by 9. Recall that squaring a number (in this case, the number (a+b))
will result in the doubling of every number inside that number’s prime box. In order for
(a+b)2’s prime box to contain two 3’s, (a+b)’s prime box must contain at least one 3.
Therefore, (a+b) is divisible by 3. Statement (1) is sufficient. Statement (2) tells us that
(a-b)2’s prime box does not contain a 3. Therefore, (a-b)’s prime box must not contain a
3. But we know nothing about (a+b). For instance, (a-b) could equal (3-1) or (2-1).
Neither of these numbers is divisible by 3, but while 3+1 = 4 is not divisible by 3, 2+1 =
3 is divisible by 3. Statement (2) is insufficient. The correct answer is A.
Problem 29 Solution: A

Simplifying the given fraction, we are asked whether 6ab/5c is a whole number.
In order for this to be true, every number in the denominator must be canceled by a
number in the numerator. Therefore, a or b must have 5 as a prime factor AND c must be
cancelled out by the remaining factors of 6ab. Statement (1) tells us that ab = 12.
Knowing all of the prime factors in the numerator, we can quickly see that 5 is not in the
numerator and will not be cancelled out of the denominator. Therefore, the fraction will
not be a whole number. Statement (1) is sufficient. Statement (2) tells us all of the
numbers in the denominator, and we can see that the 3 in the denominator can be
cancelled by the three in the numerator. But without knowing what ab is, we are unable
to see whether the 5 in the denominator can be cancelled. Statement (2) is insufficient.
The correct answer is A.

Problem 30 Solution: B

Given a complicated equation, we start by simplifying:

2ab + 10a = 50 – 2b2


ab + 5a = 25 – b2 (divide both sides by 2)
a(b+5) = 25 – b2 (factor the left)
a(b+5) = (5-b)(5+b) (factor the right)
a = 5-b (divide both sides by b+5, which we can do because we know
b ≠ -5 and we are not dividing by 0)
a+b=5 (add b to both sides)

Since (a+b) = 5, (a+b)2 = 25 and 25 has three factors: 1, 5, 25. The correct answer is B.

Problem 31 Solution: C

Pattern problems on the GMAT often include a very large series of items that
would be impossible (or at least unwise) to write out on paper. Instead, this problem
requires a person to recognize the pattern that repeats itself. In this case, after every 4th
car, the pattern repeats. By dividing 463 by 4, you find that there will be 115 sets of
painted cars – red, blue, black, gray. The key to solving these problems is the remainder.
Because there are 3 cars left over, the first such car will be painted red, the second will be
painted blue, and the third will be painted black. The correct answer is C.
Problem 32 Solution: D

Here is a second pattern problem where the answer again depends on the
remainder of a division problem. It would be unwise to count out 100 days and write out
what day of the week each will fall on. Instead, recognize that the 7th day after the initial
deposit would be Tuesday, as would the 14th day, the 21st day, etc. Dividing 100 by 7
tells us that there are 14 full weeks comprising 98 days. With the two leftover days, think
about where they would fall. The first day after the deposit would be a Wednesday, as
would the first day after waiting 98 days. The second day after the deposit would be a
Thursday as would the 100th day. The correct answer is D.

Problem 33 Solution: A

When dividing by 6 the possible remainders are 0 through 5. Consecutive


integers will have consecutively larger remainders when divided by the same number,
until the sequence “resets” to zero. Notice that the smallest possible sum of seven
consecutive remainders would be 0+1+2+3+4+5+0=15 and the largest possible sum
would be 5+0+1+2+3+4+5=20. Statement (1) tells us that the smallest remainder must
be divisible by 5. Since 5 is the only possible remainder that is greater than 0 and
divisible by 5, we know that the sequence of remainders must be 5+0+1+2+3+4+5=20.
Statement (1) is sufficient. Statement (2) tells us that the middle remainder must be
greater than 0 and divisible by 2. This means the middle remainder could be 2 or 4.
However, these two sequences of remainders would be different. Statement (2) is
insufficient. The correct answer is A.

Problem 34 Solution: B

On a difficult problem like this, it helps to rephrase the question to understand


how the statements would be beneficial (or not). The question gives us an equation that
is equal to 20h and asks us if h is an integer. If we divide both sides of the equation by
20, the question becomes this: if h = (n4 – 5n2 + 4)/20, is h an integer? In other words, is
(n4 – 5n2 + 4) divisible by 20? Breaking this down further, the question becomes: can we
divide (n4 – 5n2 + 4) by 2 2 5? This is a good start, but before we have any chance of
answering the question, we need to simplify the polynomial. (n4 – 5n2 + 4) is equal to (n2
– 4)(n2 – 1). These are two special products of the form (a+b)(a-b) = a2 – b2. (n2 – 4)(n2
– 1) = (n + 2)(n – 2)(n + 1)(n - 1) which can be rearranged from smallest to largest as
follows: (n – 2)(n – 1)(n + 1)(n + 2). Notice that if n were included, this would be a
series of 5 consecutive integers. In a series of five consecutive integers, if you take out
the middle number, two of the numbers will be divisible by 2, or even. (Possible patterns
are (E)(O)(E)(O)(E) or (O)(E)(O)(E)(O).) Therefore, the product of the other four
numbers must be divisible by 4. In a series of five consecutive integers, one of the five
numbers must be divisible by 5. We are trying to discover whether n is the integer
divisible by 5. If not, one of the other four numbers must be divisible by 5. The question
can then be simplified to this: Is the missing number (n) divisible by 5? Statement (1)
says that n/4 is an integer or that n is divisible by 4. We already know that the expression
must be divisible by 4. This statement tells us nothing about whether n is divisible by 5.
Therefore, it is insufficient. Statement (2) says that n/5 is an integer or that n is divisible
by 5. If n is divisible by 5, then none of (n – 2), (n – 1), (n + 1), or (n + 2) can be
divisible by 5 and the statement is sufficient to tell us that h is not an integer. The correct
answer is B.

Problem 35 Solution: D

This is a great example of a problem that can best be understood by writing out a
few examples and noticing the pattern that occurs. Notice that the problem stipulates that
the numerator must be larger than the denominator. 8!/5! = 8 7 6, 12!/7! =
12 11 10 9 8, etc. Recognize that the k!/n! will result in an integer and that there
will be (k-n) factors that end up in the numerator. The question becomes this: of the
numbers that are left over in the numerator, are they divisible by 30, or divisible by
2 3 5? Statement (1) tells us that the numbers must add up to be 30. Writing out some
examples, we see that 16!/14! = 16 15, 17!/13! = 17 16 15 14, etc. If you continue
this pattern, you will notice that the numbers 16 and 15 must be a part of the leftover
numerator. Since 15 is divisible by 3 5 and 16 is divisible by 2, 16 15 times any other
integer must be divisible by 30. Statement (1) is sufficient. Statement (2) tells us that k -
n = 6, or in other words, k must be 6 larger than n. This tells us that there will be 6
consecutive integers leftover in the numerator (eg. 12!/6! = 12 11 10 9 8 7). Our
knowledge of consecutive integers tells us that when there are 2 consecutive integers, one
must be divisible by 2, when there are 3 consecutive integers, one must be divisible by 3,
when there are 5 consecutive integers, one must be divisible by 5, etc. Since we have 6
consecutive integers multiplied together, one of the 6 integers must be divisible by 2, one
must be divisible by 3, and one must be divisible by 5. Statement (2) is sufficient. The
correct answer is D.

Problem 36 Solution: D

Recall that division problems can be interpreted as follows: dividend = divisor 


quotient + remainder. In this problem, one is dividing x by 7, or putting the number of
items into groups of 7. After the items are distributed, there are 3 things left over, the
remainder. This means that the value of x must be some number that is 3 larger than a
multiple of 7: 3, 10, 17, 24, etc. When you divide x by 7, you will get a whole number
(quotient) with a remainder of 3. The only answer choice that is 3 larger than a multiple
of 7 is 52. The correct answer is D.
Problem 37 Solution: E

ab
Start by rearranging the equation:  3.7 is equivalent to ab = 3.7(c+d) or ab
cd
= 3(c+d) + (7/10)(c+d). Since all four variables are integers, we look at each part of this
equation: ab must be a positive integer and 3(c+d) must be a positive integer, but
(7/10)(c+d) is only a positive integer if c+d is a multiple of 10. This demonstrates that
statement II is always true. Plugging in multiples of 10 for c+d, we will find that ab must
be a multiple of 37: 37, 74, 111, etc. This demonstrates that statement I is not always
true. Finally, if we consider that c+d is a multiple of 10, we realize that c+d must be an
even number. Therefore, if c is even, d must be even, and statement III is always true.
The correct answer is E.

Problem 38 Solution: D

The question asks us to find the remainder when (x+y)2 is divided by 5.


Statement (1) tells us that (x+y)/5 has a remainder of 3. Therefore, (x+y)(x+y) will result
in two products that will each have a remainder of 3. When these numbers are
multiplied, the remainders of 3 will also be multiplied, leaving a remainder of 9. Since 9
is larger than 5, we subtract 5 to get a remainder of 4. To see this work in practice, you
can select a number that is three more than a multiple of 5, square that number, divide by
5 and find the remainder. 32 = 9, 82 = 64, 132 = 169. When 9, 64, or 169 is divided by 5,
the remainder is 4. Statement (1) is sufficient. Statement (2) tells us that both x and y
have a remainder of 4 when each is divided by 5. By adding the two remainders, the
remainder of (x+y)/5 would be 8, but we need to subtract 5 to get a remainder of 3. Then
the remainder of 3 is squared, giving us a remainder of 9, from which we subtract 5 to get
a remainder of 4. Again, we can also choose numbers to substitute for x and y: (4+4)2 =
64, (4+9)2 = 169, (9+14)2 = 529. When 64, 169, or 529 is divided by 5, the remainder is
4. Statement (2) is sufficient. The correct answer is D.

Problem 39 Solution: E

Setting up the information in the question in the form of an equation, we see that:

x/10 = y +4/10 or x = 10y + 4

Plugging in any positive integer value for y, we see that x can be 14, 24, 34, etc. Next we
divide this equation by 5

x = 10y + 4
x/5 = 2y + 4/5
We see that 2y will still give us an integer and the remainder becomes 4. Alternatively,
dividing 4, 14, 24, 34, etc. by 5, we see that there will always be a leftover value of 4.
The correct answer is E.

Problem 40 Solution: A

Setting up the information in the question in the form of an equation, we see that:

x/13 = y + 3/13
x = 13y +3

and

x/7 = z + 3/7
x = 7z + 3

Setting the two values for x equal to one another we see that

13y +3 = 7z +3
13y = 7z

Because y and z must be whole numbers, y must have 7 as a factor and z must
have 13 as a factor. y and z can share an unlimited number of factors, but y must have a 7
in its prime box and z must have a 13 in its prime box.

yz
The question now asks what is the remainder of . Since 13 is in the
13
numerator, it can be cancelled out of the fraction, leaving a 1 in the denominator and
resulting in a whole number which has a remainder of 0. The correct answer is A.

You might also like